Discussion

The reasoning in the argument is most vulnerable to criticism on the grounds that the argument
(A)fails to consider whether corporations that do not currently use motivational posters would increase their employees’ motivation to work productively if they began using the posters
(B)...
(C)...
(D)...
(E)...
(F)...
*This question is included in Free Complete Section: LR-B, June '07 LSAT, question #4

The solution is

Posted: 05/18/2012 09:30
Someone explain why it is E not D
Posted: 05/19/2012 19:53
Choice E is the one most directly
critical of the argument. Choice D is a bit speculative.
Posted: 07/30/2012 02:58
Could anyone please tell me why it's not A.
Image Not Available
Contributor
Posted: 08/01/2012 21:17
Aristotle -

The basic problem with answer [A] is that it concerns only "corporations that do not currently use motivational posters"; it is therefore irrelevant to the argument, which does not concern such corporations in any way.

To see this, notice that sentence 1 concerns only corporations that use the posters, and that sentences 2 and 3 refer to "these corporations"; i.e., all three sentences concern the same set of corporations, and there is no attempt to generalize to any others. Thus any observation about "corporations that do not currently use motivational posters" is irrelevant to the argument. Answer [E], however, does apply to the corporations of the original argument (those that do use motivational posters, most of whose employees are already motivated).

If you still have questions about this problem, please don't hesitate to post again!

Best,
Lyn
Posted: 08/01/2012 18:24
Aristotle, in this type of LSAT question, we should look for an answer that will most weaken (or strengthen -- depending on the question) the argument being stated. In this case, we are asked which will most weaken the main argument, which is:

"The use of motivational posters is unlikely to increase productivity, since almost all employees are already motivated to work productively."

Compare A to E (summarized below for easier comparison)

A says: "beginning the use of motivational posters would increase employees' motivation"

E says: "even if a company's employees are already motivated, using motivational posters would increase that motivation"

E is stronger than A, since A says nothing about where the employees' current motivation is, whereas E takes the existing motivation to a higher level, even if it's already high to begin with.
Posted: 08/02/2012 20:09
Thanks for replying so fast and explaining the question and answers.
Posted: 08/02/2012 20:14
Aristotle, you are welcome. If you like this app, please show it some love and rate it in the App Store. The only way we can sustain the great service is if others know about it.
Posted: 11/16/2012 22:18
D may be speculative, but it's certainly relevant. Isn't the premise that "almost all employees at these corporations are already motivated to work productively" a hasty generalization?

Have you ever worked at a corporation? Yeah. . .that's definitely a hasty generalization.
Posted: 02/07/2013 09:46
Why is the answer E?

You need to be signed in to perform that action.

Sign In